Understanding Riemann Sums and Limits | Homework #16 Question

Click For Summary
The discussion focuses on understanding Riemann sums and their relationship to definite integrals. The user seeks clarification on the expression for a Riemann integral and how to represent it using limits and summation formulas. They provide an initial attempt at the expression, indicating uncertainty about the components of the sum, specifically identifying f(x) and dx. The conversation emphasizes the need to define these elements clearly to understand the transition from Riemann sums to integrals. Overall, the thread aims to clarify the formulation of Riemann sums in the context of calculus.
lLovePhysics
Messages
169
Reaction score
0

Homework Statement


Question regarding #16

III-16.jpg



Homework Equations



Riemman Sum

The Attempt at a Solution



I know that the limit of the Riemman Sum is basically the integral. However, I do not know where to go from there. Do I need to use the Summation formulas? Thanks
 
Physics news on Phys.org
can you write down the expression for a Riemann integral. something like:

\int_a^b f(x) dx = \lim_{N \to \infty} \sum_{n=1}^N ?

what goes in the question marks?

also, even though Riemann doesn't, assume things in the ? are equally spaced. that's usually good enough.
 
Last edited:
rbj said:
can you write down the expression for a Riemann integral. something like:

\int_a^b f(x) dx = \lim_{N \to \infty} \sum_{n=1}^N ?

what goes in the question marks?

also, even though Riemann doesn't, assume things in the ? are equally spaced. that's usually good enough.

Well, here's what I have so far:

\int_a^b f(x) dx = \lim_{N \to \infty} \sum_{n=1}^N (\frac{1}{N}\sin(\frac{\pi i}{N})
 
So what part of that sum is dx and what part is f(x)? After you know f(x), what is its integral?
 
Question: A clock's minute hand has length 4 and its hour hand has length 3. What is the distance between the tips at the moment when it is increasing most rapidly?(Putnam Exam Question) Answer: Making assumption that both the hands moves at constant angular velocities, the answer is ## \sqrt{7} .## But don't you think this assumption is somewhat doubtful and wrong?

Similar threads

  • · Replies 3 ·
Replies
3
Views
1K
  • · Replies 2 ·
Replies
2
Views
2K
  • · Replies 3 ·
Replies
3
Views
2K
  • · Replies 3 ·
Replies
3
Views
2K
  • · Replies 7 ·
Replies
7
Views
1K
  • · Replies 6 ·
Replies
6
Views
2K
  • · Replies 4 ·
Replies
4
Views
2K
  • · Replies 8 ·
Replies
8
Views
2K
Replies
11
Views
2K
  • · Replies 2 ·
Replies
2
Views
2K